Show that a matrix has a right inverse

  • Thread starter Thread starter Mr Davis 97
  • Start date Start date
  • Tags Tags
    Inverse Matrix
Click For Summary
A matrix A with rank m implies it has full row rank, allowing for the existence of a right inverse matrix B such that AB = I_m. The discussion highlights that surjectivity of the linear map L_A: R^n → R^m is essential for establishing the existence of a right inverse, although it is noted that surjectivity must be proven rather than assumed. The Rank-Nullity theorem is referenced, indicating that if n > m, the nullity of A is positive, which supports the existence of right inverses. Techniques such as Gram-Schmidt factorization and Singular Value Decomposition are suggested as methods to derive the right inverse. The conversation emphasizes the importance of understanding the properties of the matrices involved to complete the proof.
Mr Davis 97
Messages
1,461
Reaction score
44

Homework Statement


Let ##A## be an ##m \times n## matrix with rank ##m##. Prove that there exists an ##n \times m## matrix ##B## such that ##AB= I_m##

Homework Equations

The Attempt at a Solution


So here is how far I get. I am given that ##A## has rank ##m##. Since ##L_A(x) = Ax## is a map ##\mathbb{R}^n \rightarrow \mathbb{R}^m##, this means that ##L_A## is a surjective map.
I know that in set theory, surjective maps must have right-inverses, so I get the sense that I am on the right track. But I am not sure how to continue the proof.
 
Physics news on Phys.org
You should be more precise in your expression. The fact that you have a map ##\mathbb R^n\to\mathbb R^m ## doesn't imply it's surjective.

What do we know about ##m, n ##? Right iverses certainly exist [in fact, there's a whole family of such right inverses] when there are more columns than rows and the matrix ##A## has a full row rank.

The Rank-Nullity theorem provides a solution.
 
nuuskur said:
You should be more precise in your expression. The fact that you have a map ##\mathbb R^n\to\mathbb R^m ## doesn't imply it's surjective.

What do we know about ##m, n ##? Right iverses certainly exist [in fact, there's a whole family of such right inverses] when there are more columns than rows and the matrix ##A## has a full row rank.

The Rank-Nullity theorem provides a solution.
The Rank-Nullity theorem seems to tell me that ##\text{nullity}(A) = n-m>0##. But I don't see how this helps me
 
my general approach is to make use of orthogonality for these things -- it is a much cleaner cut than general linear independence. Since we are in reals, consider using gram-schmidt and get:

##\mathbf A = \mathbf{T V}^T##

(this is an atypical factorization but one I used in another of your threads recently), or more typically:

##\mathbf A^T = \mathbf{Q R}##, hence ##\mathbf A = \Big(\mathbf R^T \mathbf Q^T \Big)##.

Where ##\mathbf Q## is orthogonal, but not full rank (it is n x m) and ##\mathbf R## is square m x m matrix.

What happens if you multiply

##\Big( \mathbf R^T \mathbf Q^T\Big) \Big(\mathbf Q\Big) = \mathbf R^T \big(\mathbf Q^T \mathbf Q\big) = \mathbf R^T##. So we need to be able to invert ##\mathbf R^T##... how do I know that ##\mathbf R^T## is invertible?

You could also use Singular Value Decomposition to get this.
 
Question: A clock's minute hand has length 4 and its hour hand has length 3. What is the distance between the tips at the moment when it is increasing most rapidly?(Putnam Exam Question) Answer: Making assumption that both the hands moves at constant angular velocities, the answer is ## \sqrt{7} .## But don't you think this assumption is somewhat doubtful and wrong?

Similar threads

Replies
5
Views
1K
  • · Replies 3 ·
Replies
3
Views
2K
Replies
15
Views
2K
Replies
3
Views
2K
  • · Replies 1 ·
Replies
1
Views
1K
  • · Replies 1 ·
Replies
1
Views
2K
Replies
3
Views
1K
  • · Replies 6 ·
Replies
6
Views
2K
  • · Replies 2 ·
Replies
2
Views
2K
Replies
4
Views
2K